Connor edits 34
pages in 8 hours. Complete the table using equivalent ratios

Answers

Answer 1

If Connor edits 34 pages in 8 hours. Using the equivalent ratios, the table has been completed.

The equivalent ratios are the ratios that has same value when we compare them. We can check two or more ratios and check whether they are equivalent or not.

Connor edits 34 pages in 8 hours

Number of pages he edits in 8 hours = 34 pages

Number of pages he edits in 1 hour = 34/8 = 17/4 pages

Number of pages he edits in 12 hours = (17/4)×12

= 51 pages

Complete the table

Hence, if Connor edits 34 pages in 8 hours, using the equivalent ratios, the table has been completed

Learn more about equivalent ratio here

brainly.com/question/13513438

#SPJ1

Connor Edits 34 Pages In 8 Hours. Complete The Table Using Equivalent Ratios

Related Questions

The telephone company offers two billing plans for local calls. Plan 1 charges $29 per month for unlimited calls and plan 2 charges $17 per month plus $0.03 per call. Use and inequality to find the number of monthly calls for which plan is more economically than plan 2

Answers

Calls greater than 400 would be more economical in plan 1 than plan 2 and the inequality is 29 < 17 + 0.03x

How to determine the more economical plan?

The given parameters are:

Plan 1

Charges per month = $29

Plan 2

Charges per month = $17Monthly rate = $0.03

Let the number of months be x.

So, we have the following equation

Plan 1: 29

Plan 2: 17 + 0.03x

For plan 1 to be more economical plan than plan 2, then we have the following inequality

Plan 1 < Plan 2

This is represented as

29 < 17 + 0.03x

Subtract 0.03x from both sides of the inequality

So, we have

0.03x > 12

Divide both sides by 0.03

x > 400

How to interpret the result in (a)?

In (a), we have

x > 400

This means that the number of calls for the plan 1 to be more economical than plan 2, then the number of calls must be greater than 400

Read more about inequality at

https://brainly.com/question/25275758

#SPJ1

What is the correct answer

Answers

The measure of angle B in triangle ABC is 158°.

What is meant by the triangle?

A triangle is a 2-dimensional closed shape with three sides, three angles, and three vertices. A triangle would be a category of polygon.

The sum of a triangle's three interior angles is always 180.The sum of any two triangle sides is always bigger than this same length of a third side.A triangle's area is equal to half the product of it's own base and height.

For the given angles of the triangle ABC.

∠BAC = 78° , ∠CAD = 46° and ∠ADB = 110°.

Now,

∠BAC = ∠BAD + ∠DAC

Put the values;

78° = ∠BAD + 46°

∠BAD = 78° - 46°

∠BAD = 32°

Now, in a triangle; the sum of all three angles is 180°.

In triangle ADB

∠ABD + ∠BAD + ∠BDA = 180°

Put the values;

∠ABD +  32° +  110°  =  180°

∠ABD = 180°  -  142°

∠ABD = 158°

Thus, the measure of angle B is 158°.

To know more about the triangle, here

https://brainly.com/question/17335144

#SPJ13

Protein bars come in a 4 pack box or a 12 pack box the 4 pack costs 7.68 and the 12 pack costs 22.32 which box is the better

Answers

Buying 12 pack is better as it is cheaper than 4 pack.

What is unitary method?

A problem can be solved using the unitary method by first determining the value of a single unit, and then multiplying that value to determine the required value. The unitary method involves determining the value of a single unit before determining the value of the necessary quantity of units.

Given Data

Protein bars come in a 4 pack box or a 12 pack box

4 pack costs 7.68 and the 12 pack costs 22.32

In 4 pack

4 pack = 7.68

1 pack = 1.92

In 12 pack,

12 pack =  22.32

1 pack = 1.86

4 pack > 12 pack

Buying 12 pack is better as it is cheaper than 4 pack.

To learn more about unitary method, visit:

https://brainly.com/question/28276953

#SPJ9

The degree of the polynomial function f(x) is 3.
The roots of the equation f(x) = 0 are -1, 0, and 4.
Which graph could be the graph of f(x)?

Answers

The graph of the polynomial function x³ - 3x² - 4x = 0 is shown if figure.

What is Polynomial function?

A mathematical expression of algebraic terms each of which consists of a constant multiplied by one or more variables raised to a nonnegative integral power.

Given that;

The degree of the polynomial function f(x) is 3.

And, The roots of the equation f(x) = 0 are -1, 0, and 4.

Now, The polynomial function are calculated as;

f (x) = 0

(x + 1) (x - 0) (x - 4) = 0

x (x + 1) (x - 4) = 0

x (x² - 4x + x - 4) = 0

x (x² - 3x - 4) = 0

x³ - 3x² - 4x = 0

Thus, The polynomial function is;

x³ - 3x² - 4x = 0

Therefore, The graph of the polynomial function x³ - 3x² - 4x = 0 is shown if figure.

Learn more about the polynomial function visit:

https://brainly.com/question/7693326

#SPJ1

what is the standard algorithm for 72 / 3​

Answers

Long division gives the answer of 24 when 72 is divided by 3.

What is long division calculator?

Long division in mathematics is a strategy for breaking down complicated division problems into a series of simpler steps. It is the approach that division-based issues are typically solved using. The dividend, the quotient, the remainder, and the divisor can all be seen in the following long division.

So as we know:

Start by arranging it so that the dividend 72 is on the right side and the divisor 3 is on the left:

 3 ⟌ 7 2  

Start by arranging it so that the dividend 72 is on the right side and the divisor 3 is on the left:

    2    

   3 ⟌ 7 2  

Write the result (3 x 2 = 6) underneath the dividend after multiplying the divisor by the outcome from the previous step.

     2    

   3 ⟌ 7 2  

       6    

Write the solution below after subtracting the result from the previous step from the dividend's first digit (7 - 6 = 1).

Reduce the dividend's second digit (2) as follows:

    2    

   3 ⟌ 7 2  

     - 6    

       1 2  

The bottom number (12) is multiplied by the divisor (3) four times (s). Put 4 thus on top

Subtract the result of the previous step (3 x 4) from the divisor to get the solution (12), which you should write at the bottom:

  2 4  

   3 ⟌ 7 2  

     - 6    

       1 2  

      1 2  

Take the number above it and deduct the outcome from the preceding calculation. Write the response at the bottom after calculating (12 - 12 = 0).

The top number is the solution, and the remaining value is the bottom one.

Therefore, the result of 72 divided by 3 using long division is 24.

To know more about long division method , click here:
https://brainly.com/question/28784606

#SPJ13

Simplify the quantity 8 minus one third times the square root of 9 end quantity squared plus the quantity 1 minus 5 end quantity squared.

Answers

The statement quantity 8 minus one third times the square root of 9 end quantity squared plus the quantity 1 minus 5 end quantity squared is simplified to 65

What are fractions?

Fractions are defined as parts of a whole element, set or number.

There are various types of fractions, which includes;

Complex fractionsProper fractionsSimple fractionsMixed fractionsImproper fractions

Based on the information provided, we have;

8 minus one third times the square root of 9 = 8 - 1/3(√9)1 minus 5 end quantity squared = (1 - 5)²

Now, substitute the values

(8 - 1/3(√9))² +  (1 - 5)²

Find the square root

(8 - 1/3(3))²+ (-4)²

Find the square

(8 - 1)² + 16

7² + 16

49 + 16

Add the values

65

Hence, the value of the expression is 65

Learn more about square root here:

https://brainly.com/question/428672

#SPJ1

Answer Please
F(x)=5x-1

Answers

Answer:

-1

Step-by-step explanation:

Correct option is A)

Given,

f(x)=5x−1

To find: f(0)

Put x=0 in the given eqation.

Therefore, we get

f(0)=5(0)−1

=0−1

=−1

Algebra 2, 50 points, include steps please
(6-4i)(1+5i)-(3-i)

Answers

The simplified representation of the complex number expression is

23 + 37i

a + bi, where a and b are real numbers, can be used to represent any complex number.

A complex number is a component of a number system that includes an element with the symbol I, sometimes known as the imaginary unit, and that extends the real numbers by satisfying the equation i² = -1. i was described as an imaginary number by René Descartes since no real number can fulfil the aforementioned equation. The complex number a + bi is known as having real and imaginary parts, respectively, A and b. The group of complex numbers is denoted by the letter C.

The complex number expression is (6-4i)(1+5i)-(3-i)

now let us simplify,

or, (6-4i)(1+5i)-(3-i)

or, 6 + 30i - 4i -20i² -3 + i

or, 6 + 36i + 20 - 3 + i (we know that i² = -1)

or, 23 + 37i

Therefore the simplified complex expression is 23 + 37i

To learn more about complex numbers visit:

https://brainly.com/question/20566728

#SPJ1

LESSON 1 SESSION
Solve.
Numbers with more than three digits have a comma to separate groups
of three digits. Digits in groups of three places are called periods.
Thousands Period
Hundred
Thousands
2
Ten
Thousands

8
Thousands Hundreds
4
Ones Period
3
Tens
7
Ones
1
Write the number shown above in standard form (the way you usually see it).

Answers

A method of notation in which digits are divided into groups of three by commas is known as the standard form of a number.

When a number has three digits, these groups are termed periods?

The periods are separated by commas.

Large numbers have three spots between each group of digits. Periods are the names of the groups.

The periods are typically separated by commas.

number follows 100000 and 100001

nonetheless, does not include decimals, negative integers, or fractions. Let's first express the quantity in numerical form, so 100,000 represents 100,000. The next whole number after 100,000 is the one obtained by adding 1 to the previous amount. As a result, 100000 + 1 Equals 100001, the following whole number.

To learn more about Digits refer to:

https://brainly.com/question/26856218

#SPJ13

The data valo showed a mouse driven on a single day by random sample of 13 student calculate the 38th and the 60th percent all of the data

Answers

To get the percentile

Step 1: Write the formula

[tex]\begin{gathered} P_i=(\frac{i(n+1)}{100})^{th} \\ \\ \text{where n = 13} \end{gathered}[/tex]

For the 38th percentile

[tex]P_{38}=(\frac{38(13+1)}{100})^{th}[/tex][tex]P_{38}=5.32^{th}\text{ number}[/tex]

This means that the 38th percentile is between the 5th and 6th number

[tex]\begin{gathered} P_{38}=5^{th}\text{ observation}+0.32\lbrack6^{th}-5^{th}\rbrack \\ P_{38}=41+0.32(43-41)=41.64 \\ P_{38}=41.64 \end{gathered}[/tex]

P38 = 41.64This means that approximately 38% of the data lie below 43, when the data are ranked

For the 60th percentile,

[tex]P_{60}=(\frac{60(13+1)}{100})^{th}[/tex]

[tex]P_{60}=8.4^{th\text{ }}n\nu mber[/tex]

6089

[tex]P_{}=8^{th}\text{ observation}+0.4\lbrack9^{th}-8^{th}\rbrack[/tex][tex]\begin{gathered} P_{60}=56+0.4(58-56) \\ P_{60}=56.8 \end{gathered}[/tex]

60

what's the value of k

Answers

Answer:Note that:

Explanations:

The sum of complementary angles = 90°

The angle shown is a right angle

Therefore:

k + 54.3 = 90

k = 90 - 54.3

k = 35.7°

I need to know which is the best estimate for rabbit population after 5 years

Answers

Answer:

[tex]\text{ B: 800 rabbits}[/tex]

Explanation:

Here, we want to estimate the rabbit population after 5 years

From the question, there was an approximate increase in the rabbit population by 150 (from 150 to 300) in a space of 2 years

Let us have an exponential relationship for this:

[tex]P=I.a^t[/tex]

Let us get the value of t

P is the current population at 300

I is the initial

a is ?

t is the number of years which is 2

Mathematically, we have it that:

[tex]\begin{gathered} 300\text{ =150 }\times a^2 \\ a^2\text{ = 2} \\ a\text{ = }\sqrt[]{2} \end{gathered}[/tex]

Now, for the 5 years tenor, we substitute 5 for t

We have that as:

[tex]\begin{gathered} P\text{ = 150}\times\text{ (}\sqrt[]{2})^5 \\ P\text{ = 849} \end{gathered}[/tex]

From the option, we have an approximate value of 800 rabbits

Find all angles, osO<360, that satisfy the equation below, to thenearest 10th of a degree.cos(0) =-1/6

Answers

The cosine of the angle given is negative.

Cosine is negative in the second and third quadrant

[tex]\begin{gathered} 0\leq x\leq360 \\ \cos (\theta)=-\frac{1}{6} \\ \theta=\cos ^{-1}(\frac{1}{6}) \\ \theta=80.41^0 \\ \theta=80.4^0\text{ (to the nearest tenth)} \\ \end{gathered}[/tex]

In the second quadrant,

[tex]\begin{gathered} 180-\theta \\ 180-80.4=99.6 \\ \theta=99.6^0 \end{gathered}[/tex]

In the third quadrant,

[tex]\begin{gathered} 180+\theta \\ 180+80.4=260.4 \\ \theta=260.4^0 \end{gathered}[/tex]

Therefore, the values that satisfy the equation are 99.6 and 260.4 degrees.

Write the equation of the line that passes through the points (0,3) and (2,-1). Put your answer in point-slope form, using fractional form for m and b.

Answers

The equation of the line that passes through the points (0, 3) and (2, - 1) in point-slope form is y - 3 = - 2 · x, that is equivalent to y = - 2 · x + 3 (m = - 2, b = 3).

How to determine the equation of a line

According to Euclidean geometry, a line can be constructed by knowing the location of two distinct points on a plane. Then, we can determin the slope of the line from the two points. First, determine the slope of the line by using the secant line formula:

m = (- 1 - 3) / (2 - 0)

m = - 4 / 2

m = - 2

Second, substitute all the variables in the equation of the line in point-slope form:

y - 3 = - 2 · x (Point-slope form)

y = - 2 · x + 3 (Slope-intercept form)

To learn more on equations of the line: https://brainly.com/question/2564656

#SPJ1

What is the answer to the question

Answers

From the right triangle, and using the definitions of the tangent and the cosine functions, we have:

[tex]\begin{gathered} \tan B=\frac{AC}{BC}=\frac{b}{a}\Rightarrow b=a\cdot\tan B...(1) \\ \\ \cos B=\frac{BC}{AB}=\frac{a}{c}\Rightarrow c=\frac{a}{\cos B}...(2) \end{gathered}[/tex]

From the problem, we identify:

[tex]\begin{gathered} B=55.7\degree \\ a=266\text{ km} \end{gathered}[/tex]

Finally, using these values, we can find b and c.

Using (1):

[tex]\begin{gathered} b=266\cdot\tan55.7\degree \\ \\ \therefore b=389.941\text{ km} \end{gathered}[/tex]

Using (2):

[tex]\begin{gathered} c=\frac{266}{\cos55.7\degree} \\ \\ \therefore c=472.028\text{ km} \end{gathered}[/tex]

Which of the following rational fractions is graphed below?

Help please!

Answers

Answer:

[tex]\textsf{A.} \quad f(x)=\dfrac{1}{x-4}[/tex]

Step-by-step explanation:

An asymptote is a line that the curve gets infinitely close to, but never touches.

A vertical asymptote occurs when the denominator of a rational function is zero.

From inspection of the given graph, there is a vertical asymptote at [tex]x=4[/tex] .  

Therefore, the denominator of the rational function must equal zero when [tex]x=4[/tex] .

Therefore, the only solution where the denominator is zero when [tex]x=4[/tex] is:

[tex]\boxed{f(x)=\dfrac{1}{x-4}}[/tex]

help me please!!!!!!11!

Answers

Answer:

[tex]y=x^2-4x-2[/tex]

Step-by-step explanation:

Standard Form of Quadratic:

[tex]y=ax^2+bx+c[/tex]

Vertex Form of Quadratic:

[tex]y=a(x-h)^2+k[/tex]

h = vertex x

k = vertex y

Think of h and k as the ordered pair (h,k).

Match this ordered pair to the vertex ordered pair.

(h,k) = (2, -6)

Plug in (h,k) into vertex form:

[tex]y = a(x-2)^2-6[/tex]

Use other point to solve for a.

[tex]3 = a(5-2)^2-6[/tex]

[tex]3=a(3)^2-6[/tex]

[tex]3=9a-6[/tex]

[tex]9=9a[/tex]

[tex]a=1[/tex]

Plug a back into vertex form and expand the expression into standard form:

[tex]y=(x-2)^2-6[/tex]

[tex]y=(x-2)(x-2)-6[/tex]

Use F.O.I.L. method to expand (x-2)(x-2).

F(first):  [tex]x*x=x^2[/tex]

O(outer):  [tex]x*-2=-2x[/tex]

I(inner): [tex]-2*x =-2x[/tex]

L(last): [tex]-2*-2=4[/tex]

[tex]y=x^2-2x-2x+4-6[/tex]

Combine like terms:

[tex]y=x^2-4x-2[/tex]

Write the equation of the line in slope-intercept form.

The line is parallel to y + x = 3 and passes through the point (-12, 0).

Please help i will give big points

Answers

Answer:

The line is parallel to y + x =3

=> y = -x+3

the slope = -1

passes (-12, 0)

the equation would be :

y-0 = -1(x+12)

y = -x -12 Y=-x+12

Turn into mx+b form.

Y=-x+3

Plug in (-12,0)

0=-12+b

b=12

y=-x+12

It is negative X, because the slope has to be the same since it is paralle

Step-by-step explanation:

Choose the most convenient method to graph the line y=-1/4x+3

Answers

The graph for the function (y = -(1/4)x + 3) is given in the attached image. See the explanation below.



How do you plot the graph of y = -(1/4)x + 3?

Step 1 Find (x,y) pairs
Step 2: plot the points

Step 3 Draw the line on the graph.

Let's look at Step 1.

Using the graph as an example, our first pair of x,y is (0, 3)

This is obtained by substituting 0 in the equation y = -(1/4)x + 3

⇒ y = -(1/4)0 + 3

y = 0 +3

y = 3, where x = 0

If we repeat this process with x = 1, x = 2, x = 3, we will get the corresponding y for case.

Step 2: Plotting the x,y pairs will give us points on the graph.

Step 3: connecting the dots will give us the same line in the graph that is attached.

Learn more about graphs:
https://brainly.com/question/25184007
#SPJ1

(05.03 MC)
Two families visited an amusement park. The first family bought 3 hot dogs and 5 bottles of waters, which totaled $20. The second family bought 6 hot dogs and 3 bottles of
waters, which totaled $33. How much did one hot dog cost?
$3
$4
$5
$6

Answers

The cost of one hotdog based on the given situation is $5

The correct answer option is Option C

How to find the cost of one hotdog?

let

Cost of hotdogs = hCost of bottle water = w

Family A:

3h + 5w = 20

Family B:

6h + 3w = 33

Solve the two equations simultaneously:

3h + 5w = 20

6h + 3w = 33

Multiply (1) by 2

6h + 10w = 40

6h + 3w = 33

Substract the equations to eliminate h

10w - 3w = 40 - 33

7w = 7

divide both sides by 7

w = 7/7

w = 1

Substitute w = 1 into

3h + 5w = 20

3h + 5(1) = 20

3h + 5 = 20

3h = 20 - 5

3h = 15

divide both sides by 3

h = 15/3

h = 5

So therefore, the cost of one hotdog and one bottle water is $5 and $1 respectively.

Read more on simultaneous equation:

https://brainly.com/question/16863577

#SPJ1

Answer: the answer is 5 dollars

Step-by-step explanation:

The table represents a quadratic function. Write an equation of the function in
standard form.
X- -3,-2,-1,0
f(x)-6,0,-2,0

WILL GIVE BRAINLIEST

Answers

Answer:

[tex]y=2x^2+4x[/tex]

Step-by-step explanation:

Given table:

[tex]\begin{array}{|c|c|c|c|c|}\cline{1-5} x & -3 & -2 & -1 & 0\\\cline{1-5} f(x) & 6 & 0 & -2 & 0\\\cline{1-5}\end{array}[/tex]

The x-intercepts of a quadratic function are when f(x) = 0.

Therefore, the x-intercepts are:  x = -2 and x = 0.

Intercept form of a quadratic equation

[tex]y=a(x-p)(x-q)[/tex]  

where:

p and q are the x-intercepts.a is some constant.

Substitute the found x-intercepts and one of the points from the table into the formula and solve for a:

[tex]\begin{aligned} y&=a(x-p)(x-q)\\\\\implies6&=a(-3-(-2))(-3-0)\\6&=a(-1)(-3)\\6&=3a\\a&=\dfrac{6}{3}\\\implies a&=2\end{aligned}[/tex]

Substitute the x-intercepts and the found value of a into the formula:

[tex]\implies y=2(x+2)(x-0)[/tex]

Expand to standard form:

[tex]\implies y=2(x+2)(x-0)[/tex]

[tex]\implies y=2x(x+2)[/tex]

[tex]\implies y=2x^2+4x[/tex]

Please help me solve both a and b of this problem.

Answers

A. The relationship between number of towers and number of customers is proportional.

B. 12 customers.

What is a Proportional Relationship?

A proportional relationship between two variables, x and y, is a relationship whereby the ratio y/x is the same all through the table of values.

Thus, the value of y/x is constant, and is referred to as the unit rate or constant of proportionality, k.

Part A:

Given the table of values, we have:

Constant of proportionality (k) = y/x = 252/5.25 = 300/6.25 = 348/7.25 = 444/9.25 = 48.

Therefore, it is a proportional relationship because it has a constant of proportionality, k = 48.

Part B:

Substitute k = 48 into y = kx (proportional relationship)

y = 48x.

The equation for the relationship is, y = 48x. Substitute y = 576 into the equation to find the value of x, number of customers:

576 = 48x

Divide both sides by 48

576/48 = 48x/48 [division property of equality]

12 = x

x = 12

The number of customers is 12.

Learn more about proportional relationship on:

https://brainly.com/question/23318486

#SPJ1

Help! Please!!!!!!!!!?

Answers

The estimates  for [tex]6\frac{1}{9}. 1\frac{5}{7}[/tex] is [tex]10\frac{10}{21}[/tex]

The actual answer of [tex]6\frac{1}{9}. 1\frac{5}{7}[/tex] is [tex]10\frac{30}{63}[/tex]

How to estimate fractions?

The estimated answer of the fraction can be done as follows:

[tex]6\frac{1}{9}. 1\frac{5}{7}[/tex]

Hence,

[tex]6\frac{1}{9} = \frac{55}{9}[/tex]

[tex]1\frac{5}{7} = \frac{12}{7}[/tex]

Therefore, the actual answer of the fractions is as follows:

[tex]\frac{55}{9}.\frac{12}{7} = \frac{660}{63} = 10\frac{30}{63}[/tex]

Hence, the estimated answer of the fractions can be calculated as follows

[tex]\frac{55}{9}.\frac{12}{7} = \frac{660}{63} = \frac{220}{21} = 10\frac{10}{21}[/tex]

learn more on fraction and estimation here: https://brainly.com/question/14720183

#SPJ1

Consider the following expression 2(x+3). Which of the following represents the correct use of the distributive property? A. 2x+3 B. 2x+6 C. X + 6 D. X^2 + 6x + 9

Answers

The answer is B 2x+6. Because 2 times x is 2x and 2 times 3 is 6.

A linear function is transformed from y = 3/4x + 1/2 to y = 3/4x + 5/2. What is the corresponding vertical change?A)The graph is shifted up 2 units,B)The graph is shifted down 2 units. C)The graph is shifted up 4 unitsD)The graph is shifted up 5/2 units

Answers

[tex]\begin{gathered} \text{Transformed from} \\ \frac{3}{4}x+\frac{1}{2} \\ to \\ \frac{3}{4}x+\frac{5}{2} \\ A\mathrm{}\text{The graph is shifted up 2 units} \\ \frac{3}{4}x+\frac{1}{2}+2=\frac{3}{4}x+\frac{5}{2} \end{gathered}[/tex]

A. The graph is shifted up 2 units

The probability that a new machine will not need any repairs within t years from now is modeled by an exponential function of t. This probability is multiplied by 0.2 whenever the time period t is extended by 3 years as shown by the function belowf(t) = (0.2)^t/3 If the probability that the machine does not need repairs right now is 1, what is the probability that the machine will not need repairs within 12 years from now, according to the model? 0.8 0.05 0.008 0.0016

Answers

viven

t = time

function

[tex]f(t)=0.2^{t/3}[/tex]

what is the probability that the machine will not need repairs within 12

rocedure

[tex]\begin{gathered} f(12)=0.2^{12/3} \\ f(12)=0.0016 \end{gathered}[/tex]

The probability would be 0.0016

8=3+2i and z9 = 4+3i.

Answers

Answer:

8 - 3 = 5

5 divided by 2 = 2.5

i = 2.5

3 x 2.5 = 7.5

4 + 7.5 = 11.5

11.5 divided by 9 = 1.277

z = 1.277

Hope this helps!

AlphaFay

Use the confidence level and sample data to find a confidence interval for estimating the population μ. Round your answer to the same number of decimal places as the sample mean.

A group of 59 randomly selected students have a mean score of 29.5 with a standard deviation of 5.2 on a placement test. What is the 90% confidence interval for the mean score, μ, of all students taking the test?

Answers

The 90% confidence interval for the mean score, μ, of all students taking the test is; CI = (28.36, 30.64)

What is the Confidence Interval?

The formula for confidence interval is;

CI = x' ± z(s/√n)

where;

CI = Confidence Interval

x' is sample mean

z is critical value at confidence level

s is standard deviation

n is sample size

We are given;

n = 59

x' = 29.5

s = 5.2

z at CL 0f 90% = 1.645

Thus;

CI = 29.5 ± 1.645(5.2/√59)

CI = 29.5 ± 1.136

CI = (28.36, 30.64)

Read more about Confidence Interval at; https://brainly.com/question/17097944

#SPJ1

||x+4|-7|<5 how to solve this?

Answers

Answer: [tex](-16, -6) \cup (-2, 8)[/tex]

Step-by-step explanation:

[tex]||x+4|-7| < 5\\\\-5 < |x+4|-7 < 5\\\\2 < |x+4| < 12\\\\1. -2 < x+4 < 2 \implies -6 < x < -2\\\\2. -12 < x+4 < 12 \implies -16 < x < 8\\\\\therefore (-16, -6) \cup (-2, 8)[/tex]

Mrs. Parkow gives her class clues
about a 5-digit mystery number,
• The 3 is in a place that is 100 times
greater than the place of the 4,
The 2 is in a place that is 10 times
less than the place of the 3.
The 8 is in a place that is 10 times
less than the place of the 4.
The 7 is in a place that is 100 times
greater than the 2


What is Mrs. Parkow's
5-digit mystery number?

Answers

Mrs. Parkow's 5-digit mystery number is 73248.

What are Numbers up to 5-Digits?

Ten thousand is the starting point for five-digit numbers, which go up to ninety-nine million, nine hundred and ninety-nine. Integers are present in the places of ones, tens, hundreds, thousands, and ten thousands in these numbers. Any positive number may be placed at these places, with the exception of 0 at the ten thousands place.

A general 5-digit number can be written as:

a×10000 + b×1000 + c×100 + d×10 + e where a, b, c, and d are single-digit numbers.

a is the ten thousands digit

b is the thousands digit

c is the hundreds digit

d is the tens digit

e is the units digit.

Let 8 be at one's place that is e

Now, it is given that 8 is in a place that is 10 times less than the place of the 4.

This implies 4 is at the ten's place that is d.

Next, it is given that 3 is in a place that is 100 times greater than the place of the 4.

That is 100×10 = 1000 place

So, 3 is at the place of b.

The 2 is in a place that is 10 times less than the place of the 3. So, 2 is at the place of c.

The 7 is in a place that is 100 times greater than the 2. That is 100×100 = 10,000 place.

Thus, 7 is at the place of a.

Thus, we obtain

a= 7

b= 3

c= 2

d= 4

e= 8

Therefore, the 5-digit mystery number is 73248.

To learn more about mystery numbers, visit:

https://brainly.com/question/24465179

#SPJ9

Other Questions
Do you believe humans evolved from apes? Why do you think that? the diagnosticSolve for n.n+45 The points on this graph represent a relationship between x- and y-values. Which statement about the relationship is true? Explain how the location of the three economic sectors on a global map reflect spatial patterns of global development. What is the mixed number and improper fraction represented in the picture shown below? ? ?? --------- and --------- ? ? Instructions:Read this article and answer the questions below:1. Write out the myths in list form. (1) If a man lifts a a 20-kgbucket from a well and does 6kJ of work, how deep is thewell? Assume the speed o thebucket remains constant as it islifted. 7 over 8 subtracted by 1 over 12 Identify a true statement of array.find(callback [, thisArg]) method.A. It tests whether the condition returned by the callback function holds for all items in array.B. It returns the value of the first element in the array that passes a test in the callback function.C. It tests whether the condition returned by the callback function holds for at least one item in array.D. It returns the index of the first element in the array that passes a test in the callback function. Apply Laws of Exponents to write an equivalent expression.d^6df^01. d^7df2. d^73. f^74. d^6f Sofia and Zara join a queue to get tickets for the school disco. There are 51 people in front of them. What are Sofia and Zara's positions in the queue? What can you infer from the speakers tone in the opening paragraphs in a legal memorandum, you have cited u.s. motors v. gen. motors europe, 559 u.s. 939 (2010). in the very next sentence, without any intervening cites, you wish to cite to page 942 of the same case. method uses a visual list of the pros and cons of buying now or waiting, buying the salesperson's product versus a competitor's, and buying versus not buying A math quiz has 3 multiple choice questions and 2 true/false questions. Each multiple choice questions has 4 answer options: A, B, C, D. Find the number of possible outcomes. list 6 geometric principles represented in the picture. In the second stanza of Old Ironsides by Oliver Wendall Holmes, the speaker of the poem is recounting the ______________events of the ship's past. 'Her deck, once red with heroes' blood,Where knelt the vanquished foe,When winds were hurrying o'er the flood,And waves were white below,No more shall feel the victor's tread,Or know the conquered knee; -The harpies of the shore shall pluckThe eagle of the sea!'A) timelyB) ingloriousC) untimelyD) glorious 4x-y=3 in slope intercept form 1. SAMPLE: Pete is driving down 7th street. He drives 150 meters in 18 seconds. Assuming he does not speed up or slow down, what is his speed in meters per second? Formula you need to use:Speed = distance / timeOR S = D / T Variable letters, measurement with unitsD = 150mT = 18sS = ? Plug measurements into formula:S = 150m / 18s Answer with units:S = ____ An adventurous archaeologist crosses between two rock cliffs by slowly going hand-over-hand along a rope stretched between the cliffs. He stops to rest at the middle of the rope. The rope will break if the tension in it exceeds 2.5510^4 N, and our hero's mass is 85.6 kg.A) If the angle between the rope and the horizontal is = 10.5, find the tension in the rope.B) What is the smallest value the angle can have if the rope is not to break?